Last visit was: 25 Apr 2024, 09:06 It is currently 25 Apr 2024, 09:06

Close
GMAT Club Daily Prep
Thank you for using the timer - this advanced tool can estimate your performance and suggest more practice questions. We have subscribed you to Daily Prep Questions via email.

Customized
for You

we will pick new questions that match your level based on your Timer History

Track
Your Progress

every week, we’ll send you an estimated GMAT score based on your performance

Practice
Pays

we will pick new questions that match your level based on your Timer History
Not interested in getting valuable practice questions and articles delivered to your email? No problem, unsubscribe here.
Close
Request Expert Reply
Confirm Cancel
SORT BY:
Date
Tags:
Show Tags
Hide Tags
VP
VP
Joined: 30 Jan 2016
Posts: 1232
Own Kudos [?]: 4560 [48]
Given Kudos: 128
Send PM
Most Helpful Reply
Veritas Prep Representative
Joined: 26 Jul 2010
Posts: 416
Own Kudos [?]: 2946 [10]
Given Kudos: 63
Send PM
General Discussion
Retired Moderator
Joined: 10 Oct 2016
Status:Long way to go!
Posts: 1144
Own Kudos [?]: 6122 [2]
Given Kudos: 65
Location: Viet Nam
Send PM
Manager
Manager
Joined: 09 Jun 2018
Posts: 170
Own Kudos [?]: 415 [2]
Given Kudos: 86
Location: United States
GMAT 1: 730 Q50 V39
GPA: 4
WE:Operations (Manufacturing)
Send PM
Re: Manager: Our company's mail-order sales have recently increased [#permalink]
2
Kudos
Argument says: Policy changes caused increase in sales (X caused Y).

Any answer saying Z caused Y or Y did not happen implies X did not happen can be correct.

Option A talks in similar lines to the latter logic above.
Intern
Intern
Joined: 18 Aug 2018
Posts: 10
Own Kudos [?]: 1 [0]
Given Kudos: 97
Location: Turkey
Concentration: Strategy, Technology
GPA: 3.31
Send PM
Re: Manager: Our company's mail-order sales have recently increased [#permalink]
I also picked A but I must admit that C was pretty tempting. Can you please explain why the option cant be C?
Board of Directors
Joined: 11 Jun 2011
Status:QA & VA Forum Moderator
Posts: 6072
Own Kudos [?]: 4689 [2]
Given Kudos: 463
Location: India
GPA: 3.5
WE:Business Development (Commercial Banking)
Send PM
Re: Manager: Our company's mail-order sales have recently increased [#permalink]
2
Kudos
elifceylan wrote:
I also picked A but I must admit that C was pretty tempting. Can you please explain why the option cant be C?


For Strengthener type questions there must be addition of new information to the existing stimulus, check carefully , option (C) is a mere restatement of the following from the passage " This increase started around the time we started offering unlimited free shipping "

Hence, (C) may be discarded.
Intern
Intern
Joined: 15 Oct 2017
Posts: 6
Own Kudos [?]: 0 [0]
Given Kudos: 8
Send PM
Re: Manager: Our company's mail-order sales have recently increased [#permalink]
In option A how can I say that the reason for decrease in mail order of other companies is definitely because they did not offer unlimited free shipping. It can also be because of other factors. Bad quality of products, damage during shipping etc.

Please help me understand this.
Manager
Manager
Joined: 29 Nov 2016
Posts: 195
Own Kudos [?]: 56 [0]
Given Kudos: 446
Location: India
GMAT 1: 750 Q50 V42
Send PM
Re: Manager: Our company's mail-order sales have recently increased [#permalink]
According to LSAT if you are able to eliminate alternate explanation then that statement is a strenthener.

What is wrong with B.

It says adveritsement was not responsible. It ineffect eliminatrd alternate explaunation.

Experts please suggest.

Posted from my mobile device
Intern
Intern
Joined: 09 Apr 2017
Posts: 7
Own Kudos [?]: 23 [2]
Given Kudos: 22
Location: India
WE:Consulting (Real Estate)
Send PM
Re: Manager: Our company's mail-order sales have recently increased [#permalink]
2
Kudos
Option E also correctly takes into consideration that since other companes are not offering free shipping on below $50, then this company is at the advantage.?
Manager
Manager
Joined: 29 Jan 2019
Posts: 62
Own Kudos [?]: 44 [1]
Given Kudos: 15
Location: India
GPA: 4
WE:Business Development (Computer Software)
Send PM
Re: Manager: Our company's mail-order sales have recently increased [#permalink]
1
Kudos
Yogesh_24 wrote:
Option E also correctly takes into consideration that since other companies are not offering free shipping on below $50, then this company is at the advantage.?


IMO: E is out of scope

there is no relation b/w sales and shipping.
Sales may have increased or decreased after charging $50 for Many Companies..we are not sure about it.

A shows the co-relation b/w sales and shipping ...much clear; Hence Correct

Thanks = KUDOS
Intern
Intern
Joined: 09 Oct 2018
Posts: 5
Own Kudos [?]: 6 [0]
Given Kudos: 48
Send PM
Re: Manager: Our company's mail-order sales have recently increased [#permalink]
IMO this should be a 600-700 level question. Passage and alternatives are pretty straight forward. Thanks.
Intern
Intern
Joined: 02 Nov 2014
Posts: 36
Own Kudos [?]: 20 [2]
Given Kudos: 20
Send PM
Re: Manager: Our company's mail-order sales have recently increased [#permalink]
2
Kudos
Abhishek009 wrote:
elifceylan wrote:
I also picked A but I must admit that C was pretty tempting. Can you please explain why the option cant be C?


For Strengthener type questions there must be addition of new information to the existing stimulus, check carefully , option (C) is a mere restatement of the following from the passage " This increase started around the time we started offering unlimited free shipping "

Hence, (C) may be discarded.


According to me C is not the right option because it talks about increase in profits while I argument talks about increase in sales. We cannot correlate the increase in profits with increase in sales nor can we say that increase in sales and profits are the same. Hence, the option itself is out of context or irrelevant to strengthen the argument
Intern
Intern
Joined: 23 May 2019
Posts: 4
Own Kudos [?]: 2 [0]
Given Kudos: 297
Send PM
Manager: Our company's mail-order sales have recently increased [#permalink]
elifceylan wrote:
I also picked A but I must admit that C was pretty tempting. Can you please explain why the option cant be C?

The question solely discusses about increase in "mail-order sales". Any discussion about profit is out of scope imo.

Posted from my mobile device
Intern
Intern
Joined: 15 Dec 2019
Posts: 9
Own Kudos [?]: 6 [1]
Given Kudos: 13
GMAT 1: 590 Q47 V23
Send PM
Re: Manager: Our company's mail-order sales have recently increased [#permalink]
1
Kudos
Hi Expert
If i am not wrong this is cause and effect type question
Option B is providing alternate solution of incresing sales. So why optoion B is not correct.
Or is it that option B is assumption we are making here
Senior Manager
Senior Manager
Joined: 31 Jan 2019
Posts: 368
Own Kudos [?]: 707 [0]
Given Kudos: 67
Location: Switzerland
Concentration: General Management
GPA: 3.9
Send PM
Manager: Our company's mail-order sales have recently increased [#permalink]
Manager: Our company's mail-order sales have recently increased 25 percent. This increase started around the time we started offering unlimited free shipping, rather than just free shipping on orders over $50. Thus, our change in policy probably caused the increase.

Which one of the following, if true, most strengthens the manager's argument?



Strengthen question

Pre-thinking

The argument says that after a change in policy there was an increase in sales and hence such increase has to be related to the change in policy.

Falsification scenario: What if the increase is related to some other X factor

Assumption: the increase is not related to any other factor than the change in policy

POE

(A) Mail-order sales have been decreasing for companies that do not offer unlimited free shipping.
Normally I would eliminate this one since it is always a stretch to compare two entities in this way (the decrease in sales might be related to other factors as well). However after eliminating all the other options this one is the best one since it allows a scenario in which the decrease in sales is related to the lack of implementation of the free delivery shipping

(B) The company did not widely advertise its change in policy.
Irrelevant

(C) The company's profits from mail-order sales have increased since the change in policy.
Profits do not remotely suggest that the change in policy is related to the increase in sales. OUT

(D) The company's change in policy occurred well after its competitors started offering unlimited free shipping.
Irrelevant

(E) Most companies offer free shipping only on mail-order purchases over $50.
Irrelevant



Intern
Intern
Joined: 15 Dec 2019
Posts: 9
Own Kudos [?]: 6 [0]
Given Kudos: 13
GMAT 1: 590 Q47 V23
Send PM
Re: Manager: Our company's mail-order sales have recently increased [#permalink]
auradediligodo wrote:
Manager: Our company's mail-order sales have recently increased 25 percent. This increase started around the time we started offering unlimited free shipping, rather than just free shipping on orders over $50. Thus, our change in policy probably caused the increase.

Which one of the following, if true, most strengthens the manager's argument?



Strengthen question

Pre-thinking

The argument says that after a change in policy there was an increase in sales and hence such increase has to be related to the change in policy.

Falsification scenario: What if the increase is related to some other X factor

Assumption: the increase is not related to any other factor than the change in policy

POE

(A) Mail-order sales have been decreasing for companies that do not offer unlimited free shipping.
Normally I would eliminate this one since it is always a stretch to compare two entities in this way (the decrease in sales might be related to other factors as well). However after eliminating all the other options this one is the best one since it allows a scenario in which the decrease in sales is related to the lack of implementation of the free delivery shipping

(B) The company did not widely advertise its change in policy.
Irrelevant

(C) The company's profits from mail-order sales have increased since the change in policy.
Profits do not remotely suggest that the change in policy is related to the increase in sales. OUT

(D) The company's change in policy occurred well after its competitors started offering unlimited free shipping.
Irrelevant

(E) Most companies offer free shipping only on mail-order purchases over $50.
Irrelevant






Hi
How is B irrelevant?
If we say company did advertise it policy of unlimited shipping then author assimption[change in policy cause increase in profit] is in question
Manager
Manager
Joined: 18 Nov 2018
Posts: 87
Own Kudos [?]: 85 [0]
Given Kudos: 42
Send PM
Re: Manager: Our company's mail-order sales have recently increased [#permalink]
The premise says X (unlijmited free shipping) is correlated with Y (sales increase) . The correlation might be a random correlation or a causation. The conclusion says X causes Y.

To prove the conclusion, we need to :
+ show that without X, Y doesn't exist
+ eliminate other potential reasons that might cause Y

The answer choice (A) proves the conclusion by showing that without X, Y doesnt exist.
Manager
Manager
Joined: 18 Nov 2018
Posts: 87
Own Kudos [?]: 85 [1]
Given Kudos: 42
Send PM
Re: Manager: Our company's mail-order sales have recently increased [#permalink]
Yogesh_24 wrote:
Option E also correctly takes into consideration that since other companes are not offering free shipping on below $50, then this company is at the advantage.?


E actually might weaken the conclusion. With E, the reason why sales increase is that other companies don't offer free shipping for order less than $50 but less that the company offer unlimited free shipping.
Senior Manager
Senior Manager
Joined: 23 Jan 2018
Posts: 255
Own Kudos [?]: 234 [0]
Given Kudos: 359
Location: India
Concentration: Technology, General Management
GMAT 1: 640 Q48 V29
GMAT 2: 700 Q49 V36 (Online)
WE:Information Technology (Computer Software)
Send PM
Re: Manager: Our company's mail-order sales have recently increased [#permalink]
The company's change in policy occurred well after its competitors started offering unlimited free shipping.
I chose B.I thought this option rules out lack of competition and it is indeed the policy change that caused the increase in the sales.

I don't choose A because this is a causal argument and although it conveys that in absence of cause effect is absent, there can be many other reasons in the other companies.
Please explain where I am wrong.

Regards,
Arup Sarkar
Senior Manager
Senior Manager
Joined: 23 Jan 2018
Posts: 255
Own Kudos [?]: 234 [0]
Given Kudos: 359
Location: India
Concentration: Technology, General Management
GMAT 1: 640 Q48 V29
GMAT 2: 700 Q49 V36 (Online)
WE:Information Technology (Computer Software)
Send PM
Re: Manager: Our company's mail-order sales have recently increased [#permalink]
The company's change in policy occurred well after its competitors started offering unlimited free shipping.
I chose B.I thought this option rules out lack of competition and it is indeed the policy change that caused the increase in the sales.

I don't choose A because this is a causal argument and although it conveys that in absence of cause effect is absent, there can be many other reasons in the other companies.
Please explain where I am wrong.

Regards,
Arup Sarkar
GMAT Club Bot
Re: Manager: Our company's mail-order sales have recently increased [#permalink]
 1   2   
Moderators:
GMAT Club Verbal Expert
6920 posts
GMAT Club Verbal Expert
238 posts
CR Forum Moderator
832 posts

Powered by phpBB © phpBB Group | Emoji artwork provided by EmojiOne